Increasing function on R that is discontinuous on the rationals











up vote
4
down vote

favorite
4












The question (Folland's Real Analysis, 3.5.30) asks to produce an increasing function on $mathbb{R}$ whose set of discontinuities is the rationals. My train of thought is as follows:



Let $f_0$ be the identity. We want to create a jump at every rational number, while keeping the function increasing. Create $f_1$ by cutting $f_0$ at each integer, and halving the slope of each resulting line segment, fixing the left endpoints. This results in a sort of slanted stair that is still increasing and discontinuous exactly at the integers. Given $f_{n-1}$, we create $f_n$ by cutting each segment of $f_{n-1}$ into $n$ parts and halving the slope of each resulting segment, again fixing the left endpoints. Note: this is equivalent to making cuts at all rational points $frac{m}{n!}$ at the $n^text{th}$ step, with $m$ and $n$ not necessarily coprime.



The limit of such sequence of functions exists. It's easy to prove that any rational point is eventually a left endpoint, and is thus kept fixed by all successive functions in the sequence. For an irrational point $x$ we can make two sequences $a_n$ and $b_n$ of rationals converging from the left and right respectively, such that $a_n=frac{m}{n!}<x$ and $b_n=frac{m+1}{n!}>x$. Given $epsilon$, we can then choose $N$ large enough with $a_N$ and $b_N$ on the same segment of $f_{N-1}$ (so the jump from $f_{N-1}(a_N)$ to $f_{N-1}(b_N)$ is small), with this jump being less than $epsilon$. Since we can pinpoint the value of $f_n(x)$ between arbitrarily close values, the limit $f(x)=lim_n f_n(x)$ exists.



It's clear that the function is discontinuous at all rationals since it is constructed to be, and it is increasing since each $f_n$ is increasing.



Can I say that $f$ is continuous at each irrational? My gut feeling is that, since any $delta$-ball around an irrational contains a rational to the left and one to the right, we cannot necessarily say $f$ is continuous there. However we know by theorem 3.23 in the book that the set of discontinuities of an increasing function is countable. Is there a contradiction somewhere? If so, can the construction be tweaked or is a different construction necessary?



Thank you.










share|cite|improve this question


















  • 1




    Your construction looks promising. A function can be continuous at $x$ even if every neighbourhood of $x$ contains a point of discontinuity (e.g., let $f(x)$ be $0$ if $x$ is rational and be $x$ if $x$ is irrational, then $f$ is continuous at $0$). Try constructing the $epsilon$-$delta$ proof that your function is continuous at irrational numbers.
    – Rob Arthan
    Nov 21 at 23:55












  • What I'm thinking is this: let $epsilon>0$, assume there exists $delta$ such that in a $delta$-ball around $x$, $|f(x)-f(y)|<epsilon$. Take two such rationals $p$ and $q$; then we have $|f(p)-f(q)|<2epsilon$ by triangle inequality. Is this a problem?
    – Luca S.
    Nov 21 at 23:56












  • I don't think so. Write the $epsilon$-$delta$ argument down and check it for yourself.
    – Rob Arthan
    Nov 22 at 0:01






  • 1




    I upvoted the question since I think your approach will work. Would you be interested in another function having the required properties? [I have one for which the properties are somewhat simple to show.]
    – coffeemath
    Nov 22 at 4:35










  • Sure, go ahead!
    – Luca S.
    Nov 22 at 7:17















up vote
4
down vote

favorite
4












The question (Folland's Real Analysis, 3.5.30) asks to produce an increasing function on $mathbb{R}$ whose set of discontinuities is the rationals. My train of thought is as follows:



Let $f_0$ be the identity. We want to create a jump at every rational number, while keeping the function increasing. Create $f_1$ by cutting $f_0$ at each integer, and halving the slope of each resulting line segment, fixing the left endpoints. This results in a sort of slanted stair that is still increasing and discontinuous exactly at the integers. Given $f_{n-1}$, we create $f_n$ by cutting each segment of $f_{n-1}$ into $n$ parts and halving the slope of each resulting segment, again fixing the left endpoints. Note: this is equivalent to making cuts at all rational points $frac{m}{n!}$ at the $n^text{th}$ step, with $m$ and $n$ not necessarily coprime.



The limit of such sequence of functions exists. It's easy to prove that any rational point is eventually a left endpoint, and is thus kept fixed by all successive functions in the sequence. For an irrational point $x$ we can make two sequences $a_n$ and $b_n$ of rationals converging from the left and right respectively, such that $a_n=frac{m}{n!}<x$ and $b_n=frac{m+1}{n!}>x$. Given $epsilon$, we can then choose $N$ large enough with $a_N$ and $b_N$ on the same segment of $f_{N-1}$ (so the jump from $f_{N-1}(a_N)$ to $f_{N-1}(b_N)$ is small), with this jump being less than $epsilon$. Since we can pinpoint the value of $f_n(x)$ between arbitrarily close values, the limit $f(x)=lim_n f_n(x)$ exists.



It's clear that the function is discontinuous at all rationals since it is constructed to be, and it is increasing since each $f_n$ is increasing.



Can I say that $f$ is continuous at each irrational? My gut feeling is that, since any $delta$-ball around an irrational contains a rational to the left and one to the right, we cannot necessarily say $f$ is continuous there. However we know by theorem 3.23 in the book that the set of discontinuities of an increasing function is countable. Is there a contradiction somewhere? If so, can the construction be tweaked or is a different construction necessary?



Thank you.










share|cite|improve this question


















  • 1




    Your construction looks promising. A function can be continuous at $x$ even if every neighbourhood of $x$ contains a point of discontinuity (e.g., let $f(x)$ be $0$ if $x$ is rational and be $x$ if $x$ is irrational, then $f$ is continuous at $0$). Try constructing the $epsilon$-$delta$ proof that your function is continuous at irrational numbers.
    – Rob Arthan
    Nov 21 at 23:55












  • What I'm thinking is this: let $epsilon>0$, assume there exists $delta$ such that in a $delta$-ball around $x$, $|f(x)-f(y)|<epsilon$. Take two such rationals $p$ and $q$; then we have $|f(p)-f(q)|<2epsilon$ by triangle inequality. Is this a problem?
    – Luca S.
    Nov 21 at 23:56












  • I don't think so. Write the $epsilon$-$delta$ argument down and check it for yourself.
    – Rob Arthan
    Nov 22 at 0:01






  • 1




    I upvoted the question since I think your approach will work. Would you be interested in another function having the required properties? [I have one for which the properties are somewhat simple to show.]
    – coffeemath
    Nov 22 at 4:35










  • Sure, go ahead!
    – Luca S.
    Nov 22 at 7:17













up vote
4
down vote

favorite
4









up vote
4
down vote

favorite
4






4





The question (Folland's Real Analysis, 3.5.30) asks to produce an increasing function on $mathbb{R}$ whose set of discontinuities is the rationals. My train of thought is as follows:



Let $f_0$ be the identity. We want to create a jump at every rational number, while keeping the function increasing. Create $f_1$ by cutting $f_0$ at each integer, and halving the slope of each resulting line segment, fixing the left endpoints. This results in a sort of slanted stair that is still increasing and discontinuous exactly at the integers. Given $f_{n-1}$, we create $f_n$ by cutting each segment of $f_{n-1}$ into $n$ parts and halving the slope of each resulting segment, again fixing the left endpoints. Note: this is equivalent to making cuts at all rational points $frac{m}{n!}$ at the $n^text{th}$ step, with $m$ and $n$ not necessarily coprime.



The limit of such sequence of functions exists. It's easy to prove that any rational point is eventually a left endpoint, and is thus kept fixed by all successive functions in the sequence. For an irrational point $x$ we can make two sequences $a_n$ and $b_n$ of rationals converging from the left and right respectively, such that $a_n=frac{m}{n!}<x$ and $b_n=frac{m+1}{n!}>x$. Given $epsilon$, we can then choose $N$ large enough with $a_N$ and $b_N$ on the same segment of $f_{N-1}$ (so the jump from $f_{N-1}(a_N)$ to $f_{N-1}(b_N)$ is small), with this jump being less than $epsilon$. Since we can pinpoint the value of $f_n(x)$ between arbitrarily close values, the limit $f(x)=lim_n f_n(x)$ exists.



It's clear that the function is discontinuous at all rationals since it is constructed to be, and it is increasing since each $f_n$ is increasing.



Can I say that $f$ is continuous at each irrational? My gut feeling is that, since any $delta$-ball around an irrational contains a rational to the left and one to the right, we cannot necessarily say $f$ is continuous there. However we know by theorem 3.23 in the book that the set of discontinuities of an increasing function is countable. Is there a contradiction somewhere? If so, can the construction be tweaked or is a different construction necessary?



Thank you.










share|cite|improve this question













The question (Folland's Real Analysis, 3.5.30) asks to produce an increasing function on $mathbb{R}$ whose set of discontinuities is the rationals. My train of thought is as follows:



Let $f_0$ be the identity. We want to create a jump at every rational number, while keeping the function increasing. Create $f_1$ by cutting $f_0$ at each integer, and halving the slope of each resulting line segment, fixing the left endpoints. This results in a sort of slanted stair that is still increasing and discontinuous exactly at the integers. Given $f_{n-1}$, we create $f_n$ by cutting each segment of $f_{n-1}$ into $n$ parts and halving the slope of each resulting segment, again fixing the left endpoints. Note: this is equivalent to making cuts at all rational points $frac{m}{n!}$ at the $n^text{th}$ step, with $m$ and $n$ not necessarily coprime.



The limit of such sequence of functions exists. It's easy to prove that any rational point is eventually a left endpoint, and is thus kept fixed by all successive functions in the sequence. For an irrational point $x$ we can make two sequences $a_n$ and $b_n$ of rationals converging from the left and right respectively, such that $a_n=frac{m}{n!}<x$ and $b_n=frac{m+1}{n!}>x$. Given $epsilon$, we can then choose $N$ large enough with $a_N$ and $b_N$ on the same segment of $f_{N-1}$ (so the jump from $f_{N-1}(a_N)$ to $f_{N-1}(b_N)$ is small), with this jump being less than $epsilon$. Since we can pinpoint the value of $f_n(x)$ between arbitrarily close values, the limit $f(x)=lim_n f_n(x)$ exists.



It's clear that the function is discontinuous at all rationals since it is constructed to be, and it is increasing since each $f_n$ is increasing.



Can I say that $f$ is continuous at each irrational? My gut feeling is that, since any $delta$-ball around an irrational contains a rational to the left and one to the right, we cannot necessarily say $f$ is continuous there. However we know by theorem 3.23 in the book that the set of discontinuities of an increasing function is countable. Is there a contradiction somewhere? If so, can the construction be tweaked or is a different construction necessary?



Thank you.







real-analysis






share|cite|improve this question













share|cite|improve this question











share|cite|improve this question




share|cite|improve this question










asked Nov 21 at 23:42









Luca S.

404




404








  • 1




    Your construction looks promising. A function can be continuous at $x$ even if every neighbourhood of $x$ contains a point of discontinuity (e.g., let $f(x)$ be $0$ if $x$ is rational and be $x$ if $x$ is irrational, then $f$ is continuous at $0$). Try constructing the $epsilon$-$delta$ proof that your function is continuous at irrational numbers.
    – Rob Arthan
    Nov 21 at 23:55












  • What I'm thinking is this: let $epsilon>0$, assume there exists $delta$ such that in a $delta$-ball around $x$, $|f(x)-f(y)|<epsilon$. Take two such rationals $p$ and $q$; then we have $|f(p)-f(q)|<2epsilon$ by triangle inequality. Is this a problem?
    – Luca S.
    Nov 21 at 23:56












  • I don't think so. Write the $epsilon$-$delta$ argument down and check it for yourself.
    – Rob Arthan
    Nov 22 at 0:01






  • 1




    I upvoted the question since I think your approach will work. Would you be interested in another function having the required properties? [I have one for which the properties are somewhat simple to show.]
    – coffeemath
    Nov 22 at 4:35










  • Sure, go ahead!
    – Luca S.
    Nov 22 at 7:17














  • 1




    Your construction looks promising. A function can be continuous at $x$ even if every neighbourhood of $x$ contains a point of discontinuity (e.g., let $f(x)$ be $0$ if $x$ is rational and be $x$ if $x$ is irrational, then $f$ is continuous at $0$). Try constructing the $epsilon$-$delta$ proof that your function is continuous at irrational numbers.
    – Rob Arthan
    Nov 21 at 23:55












  • What I'm thinking is this: let $epsilon>0$, assume there exists $delta$ such that in a $delta$-ball around $x$, $|f(x)-f(y)|<epsilon$. Take two such rationals $p$ and $q$; then we have $|f(p)-f(q)|<2epsilon$ by triangle inequality. Is this a problem?
    – Luca S.
    Nov 21 at 23:56












  • I don't think so. Write the $epsilon$-$delta$ argument down and check it for yourself.
    – Rob Arthan
    Nov 22 at 0:01






  • 1




    I upvoted the question since I think your approach will work. Would you be interested in another function having the required properties? [I have one for which the properties are somewhat simple to show.]
    – coffeemath
    Nov 22 at 4:35










  • Sure, go ahead!
    – Luca S.
    Nov 22 at 7:17








1




1




Your construction looks promising. A function can be continuous at $x$ even if every neighbourhood of $x$ contains a point of discontinuity (e.g., let $f(x)$ be $0$ if $x$ is rational and be $x$ if $x$ is irrational, then $f$ is continuous at $0$). Try constructing the $epsilon$-$delta$ proof that your function is continuous at irrational numbers.
– Rob Arthan
Nov 21 at 23:55






Your construction looks promising. A function can be continuous at $x$ even if every neighbourhood of $x$ contains a point of discontinuity (e.g., let $f(x)$ be $0$ if $x$ is rational and be $x$ if $x$ is irrational, then $f$ is continuous at $0$). Try constructing the $epsilon$-$delta$ proof that your function is continuous at irrational numbers.
– Rob Arthan
Nov 21 at 23:55














What I'm thinking is this: let $epsilon>0$, assume there exists $delta$ such that in a $delta$-ball around $x$, $|f(x)-f(y)|<epsilon$. Take two such rationals $p$ and $q$; then we have $|f(p)-f(q)|<2epsilon$ by triangle inequality. Is this a problem?
– Luca S.
Nov 21 at 23:56






What I'm thinking is this: let $epsilon>0$, assume there exists $delta$ such that in a $delta$-ball around $x$, $|f(x)-f(y)|<epsilon$. Take two such rationals $p$ and $q$; then we have $|f(p)-f(q)|<2epsilon$ by triangle inequality. Is this a problem?
– Luca S.
Nov 21 at 23:56














I don't think so. Write the $epsilon$-$delta$ argument down and check it for yourself.
– Rob Arthan
Nov 22 at 0:01




I don't think so. Write the $epsilon$-$delta$ argument down and check it for yourself.
– Rob Arthan
Nov 22 at 0:01




1




1




I upvoted the question since I think your approach will work. Would you be interested in another function having the required properties? [I have one for which the properties are somewhat simple to show.]
– coffeemath
Nov 22 at 4:35




I upvoted the question since I think your approach will work. Would you be interested in another function having the required properties? [I have one for which the properties are somewhat simple to show.]
– coffeemath
Nov 22 at 4:35












Sure, go ahead!
– Luca S.
Nov 22 at 7:17




Sure, go ahead!
– Luca S.
Nov 22 at 7:17










1 Answer
1






active

oldest

votes

















up vote
4
down vote



accepted










There is a simpler construction:



If we take a numeration $(q_n)_{n in mathbb{N}}$ of $mathbb{Q}$, we can define
$$f(x) := sum_{k=1}^infty frac{1}{2^k} 1_{[q_k, infty)}(x).$$
The convergence is uniform and this function is monotone increasing by construction. Since all $1_{[q_n,infty)}$ are continuous in all points of $mathbb{R} setminus mathbb{Q}$, the limes is this too. (If $x$ is irrational, then we can take $varepsilon >0$ so small that $(x-varepsilon,x+varepsilon)$ doesn't contain the points $q_1,ldots,q_n$. So $|f(x)-f(y)| le sum_{k=n}^infty 2^{-k} le 2^{1-n}$.)



Fix $n$. For any $max_{q_i < q_n,i=1,ldots n} q_i < x < q_n < y < max_{q_i > q_n,i =1 ldots, n}$ q_i we have
$f(x) + 2^{-n} le f(y)$. Thus $f$ is discontinuous in any point of $mathbb{Q}$.






share|cite|improve this answer





















  • I'm having such a hard time visualizing a function constructed like this. The proof is sound though.
    – Rchn
    Nov 22 at 10:38










  • In fact, you cannot really visualize this function. The key point is that we only add a jump of height $2^{-n}$ in every step. Define $g(x) = sum_{k=1}^infty frac{1}{2^k} 1_{[1/k,infty)}$. This function has in $x=0$ the same behaviour (and can be visualized): In every neighboorhoud $(-varepsilon,varepsilon)$ we have infinite many jump-discontinuities. However, $g$ is continuous in $x=0$!
    – p4sch
    Nov 22 at 20:49













Your Answer





StackExchange.ifUsing("editor", function () {
return StackExchange.using("mathjaxEditing", function () {
StackExchange.MarkdownEditor.creationCallbacks.add(function (editor, postfix) {
StackExchange.mathjaxEditing.prepareWmdForMathJax(editor, postfix, [["$", "$"], ["\\(","\\)"]]);
});
});
}, "mathjax-editing");

StackExchange.ready(function() {
var channelOptions = {
tags: "".split(" "),
id: "69"
};
initTagRenderer("".split(" "), "".split(" "), channelOptions);

StackExchange.using("externalEditor", function() {
// Have to fire editor after snippets, if snippets enabled
if (StackExchange.settings.snippets.snippetsEnabled) {
StackExchange.using("snippets", function() {
createEditor();
});
}
else {
createEditor();
}
});

function createEditor() {
StackExchange.prepareEditor({
heartbeatType: 'answer',
convertImagesToLinks: true,
noModals: true,
showLowRepImageUploadWarning: true,
reputationToPostImages: 10,
bindNavPrevention: true,
postfix: "",
imageUploader: {
brandingHtml: "Powered by u003ca class="icon-imgur-white" href="https://imgur.com/"u003eu003c/au003e",
contentPolicyHtml: "User contributions licensed under u003ca href="https://creativecommons.org/licenses/by-sa/3.0/"u003ecc by-sa 3.0 with attribution requiredu003c/au003e u003ca href="https://stackoverflow.com/legal/content-policy"u003e(content policy)u003c/au003e",
allowUrls: true
},
noCode: true, onDemand: true,
discardSelector: ".discard-answer"
,immediatelyShowMarkdownHelp:true
});


}
});














draft saved

draft discarded


















StackExchange.ready(
function () {
StackExchange.openid.initPostLogin('.new-post-login', 'https%3a%2f%2fmath.stackexchange.com%2fquestions%2f3008548%2fincreasing-function-on-r-that-is-discontinuous-on-the-rationals%23new-answer', 'question_page');
}
);

Post as a guest















Required, but never shown

























1 Answer
1






active

oldest

votes








1 Answer
1






active

oldest

votes









active

oldest

votes






active

oldest

votes








up vote
4
down vote



accepted










There is a simpler construction:



If we take a numeration $(q_n)_{n in mathbb{N}}$ of $mathbb{Q}$, we can define
$$f(x) := sum_{k=1}^infty frac{1}{2^k} 1_{[q_k, infty)}(x).$$
The convergence is uniform and this function is monotone increasing by construction. Since all $1_{[q_n,infty)}$ are continuous in all points of $mathbb{R} setminus mathbb{Q}$, the limes is this too. (If $x$ is irrational, then we can take $varepsilon >0$ so small that $(x-varepsilon,x+varepsilon)$ doesn't contain the points $q_1,ldots,q_n$. So $|f(x)-f(y)| le sum_{k=n}^infty 2^{-k} le 2^{1-n}$.)



Fix $n$. For any $max_{q_i < q_n,i=1,ldots n} q_i < x < q_n < y < max_{q_i > q_n,i =1 ldots, n}$ q_i we have
$f(x) + 2^{-n} le f(y)$. Thus $f$ is discontinuous in any point of $mathbb{Q}$.






share|cite|improve this answer





















  • I'm having such a hard time visualizing a function constructed like this. The proof is sound though.
    – Rchn
    Nov 22 at 10:38










  • In fact, you cannot really visualize this function. The key point is that we only add a jump of height $2^{-n}$ in every step. Define $g(x) = sum_{k=1}^infty frac{1}{2^k} 1_{[1/k,infty)}$. This function has in $x=0$ the same behaviour (and can be visualized): In every neighboorhoud $(-varepsilon,varepsilon)$ we have infinite many jump-discontinuities. However, $g$ is continuous in $x=0$!
    – p4sch
    Nov 22 at 20:49

















up vote
4
down vote



accepted










There is a simpler construction:



If we take a numeration $(q_n)_{n in mathbb{N}}$ of $mathbb{Q}$, we can define
$$f(x) := sum_{k=1}^infty frac{1}{2^k} 1_{[q_k, infty)}(x).$$
The convergence is uniform and this function is monotone increasing by construction. Since all $1_{[q_n,infty)}$ are continuous in all points of $mathbb{R} setminus mathbb{Q}$, the limes is this too. (If $x$ is irrational, then we can take $varepsilon >0$ so small that $(x-varepsilon,x+varepsilon)$ doesn't contain the points $q_1,ldots,q_n$. So $|f(x)-f(y)| le sum_{k=n}^infty 2^{-k} le 2^{1-n}$.)



Fix $n$. For any $max_{q_i < q_n,i=1,ldots n} q_i < x < q_n < y < max_{q_i > q_n,i =1 ldots, n}$ q_i we have
$f(x) + 2^{-n} le f(y)$. Thus $f$ is discontinuous in any point of $mathbb{Q}$.






share|cite|improve this answer





















  • I'm having such a hard time visualizing a function constructed like this. The proof is sound though.
    – Rchn
    Nov 22 at 10:38










  • In fact, you cannot really visualize this function. The key point is that we only add a jump of height $2^{-n}$ in every step. Define $g(x) = sum_{k=1}^infty frac{1}{2^k} 1_{[1/k,infty)}$. This function has in $x=0$ the same behaviour (and can be visualized): In every neighboorhoud $(-varepsilon,varepsilon)$ we have infinite many jump-discontinuities. However, $g$ is continuous in $x=0$!
    – p4sch
    Nov 22 at 20:49















up vote
4
down vote



accepted







up vote
4
down vote



accepted






There is a simpler construction:



If we take a numeration $(q_n)_{n in mathbb{N}}$ of $mathbb{Q}$, we can define
$$f(x) := sum_{k=1}^infty frac{1}{2^k} 1_{[q_k, infty)}(x).$$
The convergence is uniform and this function is monotone increasing by construction. Since all $1_{[q_n,infty)}$ are continuous in all points of $mathbb{R} setminus mathbb{Q}$, the limes is this too. (If $x$ is irrational, then we can take $varepsilon >0$ so small that $(x-varepsilon,x+varepsilon)$ doesn't contain the points $q_1,ldots,q_n$. So $|f(x)-f(y)| le sum_{k=n}^infty 2^{-k} le 2^{1-n}$.)



Fix $n$. For any $max_{q_i < q_n,i=1,ldots n} q_i < x < q_n < y < max_{q_i > q_n,i =1 ldots, n}$ q_i we have
$f(x) + 2^{-n} le f(y)$. Thus $f$ is discontinuous in any point of $mathbb{Q}$.






share|cite|improve this answer












There is a simpler construction:



If we take a numeration $(q_n)_{n in mathbb{N}}$ of $mathbb{Q}$, we can define
$$f(x) := sum_{k=1}^infty frac{1}{2^k} 1_{[q_k, infty)}(x).$$
The convergence is uniform and this function is monotone increasing by construction. Since all $1_{[q_n,infty)}$ are continuous in all points of $mathbb{R} setminus mathbb{Q}$, the limes is this too. (If $x$ is irrational, then we can take $varepsilon >0$ so small that $(x-varepsilon,x+varepsilon)$ doesn't contain the points $q_1,ldots,q_n$. So $|f(x)-f(y)| le sum_{k=n}^infty 2^{-k} le 2^{1-n}$.)



Fix $n$. For any $max_{q_i < q_n,i=1,ldots n} q_i < x < q_n < y < max_{q_i > q_n,i =1 ldots, n}$ q_i we have
$f(x) + 2^{-n} le f(y)$. Thus $f$ is discontinuous in any point of $mathbb{Q}$.







share|cite|improve this answer












share|cite|improve this answer



share|cite|improve this answer










answered Nov 22 at 10:09









p4sch

4,800217




4,800217












  • I'm having such a hard time visualizing a function constructed like this. The proof is sound though.
    – Rchn
    Nov 22 at 10:38










  • In fact, you cannot really visualize this function. The key point is that we only add a jump of height $2^{-n}$ in every step. Define $g(x) = sum_{k=1}^infty frac{1}{2^k} 1_{[1/k,infty)}$. This function has in $x=0$ the same behaviour (and can be visualized): In every neighboorhoud $(-varepsilon,varepsilon)$ we have infinite many jump-discontinuities. However, $g$ is continuous in $x=0$!
    – p4sch
    Nov 22 at 20:49




















  • I'm having such a hard time visualizing a function constructed like this. The proof is sound though.
    – Rchn
    Nov 22 at 10:38










  • In fact, you cannot really visualize this function. The key point is that we only add a jump of height $2^{-n}$ in every step. Define $g(x) = sum_{k=1}^infty frac{1}{2^k} 1_{[1/k,infty)}$. This function has in $x=0$ the same behaviour (and can be visualized): In every neighboorhoud $(-varepsilon,varepsilon)$ we have infinite many jump-discontinuities. However, $g$ is continuous in $x=0$!
    – p4sch
    Nov 22 at 20:49


















I'm having such a hard time visualizing a function constructed like this. The proof is sound though.
– Rchn
Nov 22 at 10:38




I'm having such a hard time visualizing a function constructed like this. The proof is sound though.
– Rchn
Nov 22 at 10:38












In fact, you cannot really visualize this function. The key point is that we only add a jump of height $2^{-n}$ in every step. Define $g(x) = sum_{k=1}^infty frac{1}{2^k} 1_{[1/k,infty)}$. This function has in $x=0$ the same behaviour (and can be visualized): In every neighboorhoud $(-varepsilon,varepsilon)$ we have infinite many jump-discontinuities. However, $g$ is continuous in $x=0$!
– p4sch
Nov 22 at 20:49






In fact, you cannot really visualize this function. The key point is that we only add a jump of height $2^{-n}$ in every step. Define $g(x) = sum_{k=1}^infty frac{1}{2^k} 1_{[1/k,infty)}$. This function has in $x=0$ the same behaviour (and can be visualized): In every neighboorhoud $(-varepsilon,varepsilon)$ we have infinite many jump-discontinuities. However, $g$ is continuous in $x=0$!
– p4sch
Nov 22 at 20:49




















draft saved

draft discarded




















































Thanks for contributing an answer to Mathematics Stack Exchange!


  • Please be sure to answer the question. Provide details and share your research!

But avoid



  • Asking for help, clarification, or responding to other answers.

  • Making statements based on opinion; back them up with references or personal experience.


Use MathJax to format equations. MathJax reference.


To learn more, see our tips on writing great answers.





Some of your past answers have not been well-received, and you're in danger of being blocked from answering.


Please pay close attention to the following guidance:


  • Please be sure to answer the question. Provide details and share your research!

But avoid



  • Asking for help, clarification, or responding to other answers.

  • Making statements based on opinion; back them up with references or personal experience.


To learn more, see our tips on writing great answers.




draft saved


draft discarded














StackExchange.ready(
function () {
StackExchange.openid.initPostLogin('.new-post-login', 'https%3a%2f%2fmath.stackexchange.com%2fquestions%2f3008548%2fincreasing-function-on-r-that-is-discontinuous-on-the-rationals%23new-answer', 'question_page');
}
);

Post as a guest















Required, but never shown





















































Required, but never shown














Required, but never shown












Required, but never shown







Required, but never shown

































Required, but never shown














Required, but never shown












Required, but never shown







Required, but never shown







Popular posts from this blog

Quarter-circle Tiles

build a pushdown automaton that recognizes the reverse language of a given pushdown automaton?

Mont Emei